You are on page 1of 40

The problems of first TA class of Math144

T eaching Assistant : Liu Zhi


Date : F riday, F eb 2, 2007
Q1. According to the journal Chemical Engineering, an important property of a fiber is its water absorbency. A random sample of 20 pieces of
cotton fiber is taken and the absorbency on each piece was measured.
The following are the absorbency values:
18.71 21.41 20.72 21.81 19.29 22.43 20.17
23.71 19.44 20.50 18.92 20.33 23.00 22.85
19.25 21.77 22.11 19.77 18.04 21.12
a). Calculate the sample mean and median for the above sample values.
b). Compute the 10% trimmed mean.
c). Do a dot plot and constuct a relative frequency histogram of data.
Solution: a). By definition, sample mean is:
Pn
xi
415.35
= 20.7675.
x = i=1 =
n
20
Because n is even, so
1
1
x = (xn/2 + xn/2+1 ) = (20.72 + 20.50) = 20.61.
2
2
b). We can compute the trimmed mean directly:
Pn
xi 18.71 23.71 23.00 18.04
xtr(10) = i=1
= 20.7431.
16
c). Do a dot plot and construct histogram are easy works, complete
them by youself please.

The problems of second TA class of Math144


T eaching Assistant : Liu Zhi
Date : F riday, F eb 9, 2007
Q1. Show that P (E F ) 1 P (E 0 ) P (F 0 ).
Proof : P (E F ) = 1 P ([E F )]0 )
= 1 P (E 0 F 0 ),
since P (E 0 F 0 ) P (E 0 ) + P (F 0 ),
so we have P (E F ) = 1 P (E 0 F 0 ) 1 P (E 0 ) P (F 0 ).

Q2. Show that P (A 4 B) = P (A) + P (B) 2P (A B).


Proof : P (A 4 B) =
=
=
=

P ([A B] [B A])
P (A B) + P (B A)
P (A) P (A B) + P (B) P (A B)
P (A) + P (B) 2P (A B).

Alternative,
P (A 4 B) =
=
=
=

P (A B) + P (B A)
P (A B 0 ) + P (B A0 )
P (A) P (A B) + P (B) P (A B)
P (A) + P (B) 2P (A B).

Q3. From a pack of 52 cards, we draw five cards at random. What is the
probability that an Ace will appear at the fifth draw?
Solution: Denoted by A the event {an ace will appear at fifth draw}.
Then
P4 P1
P (A) = 515 4 = 0.0769231.

P52
Q4. Two dice are thrown, let E be the event that the sum of the dice is
odd, let F be the event that the first die lands on 1. And let G be the
event that the sum is 5. Describe the events E F, E F, F G, E
F 0 , E F G.
Answer: Because

E = {The sum of the dice is odd},


F = {First die lands on 1},
G = {Sum of the dice is 5}.
2

Then,
EF =
=
EF =
EG =
E F0 =
EF G =
=

{The sum of the dice is odd and first die lands on 1}


{The first die lands on 1 and second die lands on a even number},
{First die lands on 1 or the sum of the dice is odd},
{The sum of dice is 5},
{The first die lands on 1 and the sum of dice is odd},
{The sum is odd and first die lands on 1 and sum of dice is 5},
{First die lands on 1 and second die lands on 4}.

Q5. Based on Q4, Find a simple expression for the events:


(a). E E 0 ,
(b). E E 0 ,
(c). (E F ) (E F 0 ),
(d). (E F ) (E 0 F ) (E F 0 ),
(e). (E F ) (F G).
Answer: (a). E E 0 = { The sum of dice is odd or isnt odd}
= { The sum of dice is a number}.
(b). E E 0 = {The sum of dice is odd and even also} = .
(c). (E F ) (E F 0 ) = { Sum of the dice is odd or first die lands on
1, sum of the dice is odd or first die dont lands on 1}
= {The sum of dice is odd} = E.
(d). (E F ) (E 0 F ) (E F 0 ) = F (E F 0 )
= {First die lands on 1, sum of dice is odd and first die dont land on 1}
= {First die lands on 1 and sum of dice is odd} = E F .
(e). (E F ) (F G) = {First die lands on 1 or sum of dice is odd,
first die lands on 1 or the sum of dice is 5 }
= {First die lands on 1 or sum of dice is 5}.

The problems of third TA class of Math144


T eaching Assistant : Liu Zhi
Date : F riday, F eb 23, 2007
Q1. Show that if A and B are independent, then
(a). A and B 0 are independent.
(b). A0 and B 0 are independent.
Proof : (a). We recall the definition of independence, we say that two
events A and B are independent if P (A B) = P (A)P (B). Then,
P (A B 0 ) =
=
=
=

P (A) P (A B)
P (A) P (A)P (B)
P (A)(1 P (B))
P (A)P (B 0 ).

so A and B 0 are independent.


(b). P (A0 B 0 ) =
=
=
=

P (B 0 ) P (AB 0 )
P (B 0 ) P (A)P (B 0 )
P (B 0 )(1 P (A))
P (A0 )P (B 0 ).

This state the independence of A0 and B 0 .

Q2. We draw cards, one at a time, at random and successively from an


ordinary deck of 52 cards with replacement. What is the probability
that an ace appears before a face card?
Solution: Denoted by A the event {an ace appears before a face card}
and Bn the event {an ace appears at nth draw, while aces and face
cards dont appear at first n 1st draw}, then
P (A) =

P (Bn )

n=1

X
4
36
=
[( )n1 ( )]
52
52
n=1

4 X 36 n1
( )
52 n=1 52

= 1/4.
4

Q3. A die is thrown as long as necessary for an ace or a 6 to turn up. Given
that no ace turned up at the first two throwns, what is the probability
that at least three throws will be necessary?
Solution: Denoted by A the event {no ace turned up at the first two
throwns}, by B the event {at least three throws will be necessary} and
by C the event {both the result of first throw and second throw are 6}.
P (B A)
P (A)
P (C)
=
P (A)
(1/6)2
=
(5/6)2
= 1/25.

P (B/A) =

Q4. In answering a question on a multiple-choice test, a student either


knows the answer or she guesses. Let p be the probability that she
know the answer and 1 p be the probability that she guesses. Assume
that a student who guesses at the answer will be correct with probability 1/m, where m is the number of multiple choice alternatives.
What is the conditional probability that a student knew the answer to
a question given that she answered it correctly?
Answer: Let A = {she know the answer},
B = {she guess},
C = {she answer correctly}.
obviously, = A B. Then the probability that we want to know is
P (A/C).
P (A C)
P (A/C) =
P (C)
P (C/A)P (A)
=
P (A C) + P (B C)
p
=
P (C/A)P (A) + P (C/B)P (B)
p
=
P (A) + m1 P (B)
mp
=
.

(m 1)p + 1

Q5. In an urn are 5 fair coins, two 2-headed coins, and four 2-tailed coins.
A coin is to be randomly selected and flipped. Compute the probability
that the coin is fair if the result was
(a). The flip was head;
(b). 2 flips were both heads. Use prior and posterior probabilities.
Answer: Let A = {The coin is fair},
B = {The coin is 2-headed},
C = {The coin is 2-tailed}.
Then prior probability is:
P (A) = 5/11, P (B) = 2/11, P (C) = 4/11.
(a). Denoted by x the number of heads appear at first flip. Then
obviously, x is 0 or 1. Now we know x = 1 by conditions offered by
question. So
f (1/A) = 1/2, f (1/B) = 1, f (1/C) = 0.
The marginal distribution of x = 1 can be calculated as:
g(1) = f (1/A)P (A) + f (1/B)P (B) + f (1/C)P (C)
= 9/22.
Hence the posterior probability of A, given x = 1, is
(A/1) =

f (1/A)P (A)
= 5/9.
g(1)

(b). Similar as (a), denoted by x the number of heads appear after 2


flips. x maybe is 0,1 or 2. by question, we know the result is 2, so
f (2/A) = 1/4, f (1/B) = 1, f (1/C) = 0.
The marginal distribution of x = 2 can be calculated as:
g(2) = f (2/A)P (A) + f (2/B)P (B) + f (2/C)P (C)
= 13/44.
Hence the posterior probability of A, given x = 2, is
(A/2) =

f (2/A)P (A)
= 5/13.
g(2)

Q6. A grocery store sells X Hundred kilograms of rice every day, where the
distribution of the random variable X is of the following form:
F (x) =
=
=
=

0
kx2
k(x2 + 12x 3)
1

if
if
if
if

x < 0;
0 x < 3;
3 x < 6;
x 6.

Suppose that this grocery stores total sales of rice do not reach 600
kilograms on any given day.
(a). Find the value of k.
(b). What is the probability that the store sells between 200 and 400
kilograms of rice next Thursday?
(c). What is the probability that the store sells over 300 kilograms of
rice next Thursday?
(d). We are given that the store sold at least 300 kilograms of rice
last Friday. What is the probability that is did not sell more than 400
kilograms on that day?
Answer: (a). Because suppose that total sales of rice do not reach 600
kilograms on any given day, in other words P (X = 6) = 0, so
P (X 6) = P (X < 6) = k(62 + 12 6 3) = 1,
This imply k =
(b).
=
=
=
=

1
.
33

P (2 X 4)
P (2 X < 3) + P (X = 3) + (3 < X 4)
k(32 22 ) + 5/11 + k((42 + 12 4 3) (32 + 12 3 3))
25/33
0.757576.

(c). P (X > 3) =
=
=
=

1 P (X 3)
1 k(32 + 12 3 3)
3/11
0.272727.

(d). P (X 4/X 3) =

P (X4,X3)
P (X3)

= 5/6.

Q7. A random variable X is called symmetric if for all real number x such
that
P (X x) = P (X x).
7

Prove that if X is symmetric, then for all t > 0, its distribution function
F satisfies the following relations:
(a). P (| X | t) = 2F (t) 1,
(b). P (| X |> t) = 2[1 F (t)],
(c). P (X = t) = F (t) + F (t) 1.
Proof : (a). Since t > 0, so
P (| X | t) =
=
=
=
=

P (t X t)
P (X t) P (X < t)
F (t) (1 P (X t))
F (t) 1 + P (X t)
2F (t) 1.

(b). Similar as (a),


P (| X |> t) =
=
=
=
(c).

P (X = t) =
=
=
=

P (X > t) + P (X < t)
1 F (t) + 1 P (X t)
2 F (t) P (X t)
2[1 F (t)].
P (X t) P (X < t)
F (t) (1 P (X t))
F (t) 1 + P (X t)
F (t) + F (t) 1.

The problems of fourth TA class of Math144


T eaching Assistant : Liu Zhi
Date : F riday, M arch 2, 2007
Q1. The probability density function of a random variable X is given by:
f (x) = p

if 1 < x < 0;

(1 x2 )

= 0

elsewhere.

(a). Calculate the value of k,


(b). Find probability distribution function of X.
Solution: (a). Because f (x) is probability density function, so
Z
f (x)dx = 1,
+

and

k
dx
1 x2
1
= k arcsinx |01 = 1

f (x)dx =
+

Hence k = 1/.
(b). Let F (x) is the probability distribution of X, then,
Z
f (y)dy
F (x) =
x

= 0,
Z x
1

1,

if x 1,
1
1
dx = (arcsinx + ), if x [1, 0],
2

1x
if x 0.

Q2. Let X denote the lifetime of a radio, in years, manufactured by a certain


company. The density function of X is given by:
1 x
e 15
15
= 0

if x 0;

f (x) =

elsewhere.

What is the probability that, of eight such radios, at least four last
more than 15 years?
9

Solution: (a). Let F (x) is the probability distribution of the lifetime


of a radio, then Z
15
1 y
F (15) =
e 15 dy = 1 1/e,
15
0
By similar calculation, we can show that P (X 15) = 1F (15) = 1/e,
denote p = P (X 15), q = P (X 15). Hence,
P (probability of eight such radios, at least four last more than 15 years)
= C84 p4 q 4 + C85 p5 q 3 + C86 p6 q 2 + C87 p7 q + C88 p8
= 0.335034.

Q3. First a point Y is selected at random from the interval (0,1). Then
another point X is selected at random from the interval (Y ,1). Find
the probability density function of X.
Solution: Let f (x) is the density function of X, G(y) the density
function of Y and h(x, y) their joint density function. Clearly, g(y) = 1
when y (0, 1) and equals 0 otherwise. Now we have following result:
h(x, y) = f (x | y)g(y)
= f (x | y)
1
=
,
1y
= 0,

if y < x < 1, y (0, 1),


otherwise

So, f (x) is the marginal distribution of X can be calculated as following:


Z
f (x) =
h(x, y)dy
Z x
1
=
dy = ln(1 x).
0 1y
Therefore,

f (x) = ln(1 x),


= 0,

if 0 < x < 1
otherwise.

Q4. Let (X, Y ) be a random point from a unit disk centered at the origin.
Find P (0 X 4/11|Y = 4/5).
Solution: Obviously, f (x | y) = 1 2 , so
2

Z
P (0 X 4/11|Y = 4/5) =
0

1y

4
11

1
q
dx
2 1 ( 45 )2

= 10/33.
10

Q5. Stores A and B, which belong to the same owner, are located in two
different towns. If the probability density function of the weekly profit
of each store, in thousands of dollars, is given by
f (x) = x/4
= 0

if 1 < x < 3,
otherwise.

and the profit of one store is independent of the other, what is the
probability that next week one store makes at least $ 500 more than
the other store?
Solution: Let F (x)R is the probability distribution of the profit of each
x
store, then F (x) = 1 4t dt = 18 (x2 1). Hence,
Z 3Z 3
P (X Y > 0.5) =
f (x | y)f (y)dxdy
1

2.5

0.5+y
3

=
1

0.5+y

xy
dxdy
44

= 0.175781.

Q6. A random variable Y with distribution function F (y) = y/5 for 0 <
y 5. Determine the conditional expectation of Y , E[Y |Y > x] given
that Y > x and 0 < x 5.
Answer: Since given 0 < x 5, calculate it directly:
Z 5
yP (Y = y | Y > x)dy
E[Y | Y > x] =
0
Z 5
yP (Y = y, Y > x)
=
dy
P (Y > x)
0
Z 5
y/5
dy
=
x 1 x/5
= (5 + x)/2.

Q7. In a given lottery, players pick six different integers between 1 and
49, the order of selection being irrelevant. The lottery commission
then select six of these numbers at random as winning numbers. A
player wins the grand prize of $1,200,000 if all six numbers that he
has selected match the winning numbers. He wins the second and
third prizes of $800 and $35, respectively, if exactly five and four of his
selected numbers match the winning numbers. What is the expected
value of the amount a player wins in one game?
11

Answer: Let X is the random variable of the amount of a player wins


in one game, Now we want to know its expect value. Denoted by A,
B and C the events that the player wins grand prize, second prize and
third prize, respectively. Then
E[X] = 1200000P (A) + 800P (B) + 35P (C)
1
C 5C 1
C 4C 2
= 1200000 6 + 800 6 6 43 + 35 6 6 43
C49
C49
C49
= 0.134475.

12

The solutions of first quiz of Math144


T eaching Assistant : Liu Zhi
Date : F riday, M arch 16, 2007
Q1. Following are the informations that we can obtain from problem,
Age group
20 24
25 29
30 34
35 39
40 44

Frequency
128
208
194
297
240

Relative frequency
0.12
0.2
0.18
0.28
0.22

Cumulative frequence
0.12
0.32
0.5
0.78
1

So, IQR = 75th percentile 25th percentile


0.75 0.5
0.25 0.12
= 35 + (39 35)
[25 + (29 25)
]
0.78 0.5
0.32 0.12
= 10.83.
Q2. Let a is the number that we want to find, then
a = C53 23 C21 (3) C11 4
= 1920.
Q3. By the basic property of density function:
Z + Z +
1 =
f (x, y)dxdy

Z + Z +
=
kx3 ey/3 dxdy
1
Z + 1
Z +
3
= k
x dx
ey/3 dy
1

3
= k( e1/3 ).
2
So k = 23 e1/3 .

13

Q4. Use the law of probability in continuous case:


Z +
P (Y < X|X = x)f (x)dx
P (Y < X) =

Z +
P (Y < x)f (x)dx
=

Z 1Z x
f (y)dyf (x)dx
=
0
0
Z 1Z x
2ydydx
=
0

1
=
.
3
Q5. According to the definition of conditional probability,
P (X > 1/2, Y > 1/2)
P (Y > 1/2)
R 1 R 2 2 xy
(x + 3 )dydx
1/2 1/2
= R2 R1
)dxdy
(x2 + xy
3
1/2 0

P (X > 1/2|Y > 1/2) =

= 43/52.

14

The problems of fifth TA class of Math144


T eaching Assistant : Liu Zhi
Date : F riday, M arch 23, 2007
Q1. Let the probability density function of a random variable X be given
by
f (x) = |x 1|,
= 0.
find E(X 2 + X).
Solution:

EX

if 0 x 2
otherwise.

x (1 x)dx +
x2 (x 1)dx
Z0 1
Z 12
=
(x2 x3 )dx +
(x3 x2 )dx

= 1/3 1/4 + 4 1/4 8/3 + 1/3


= 3/2.
Z 1
Z 2
EX =
x(1 x)dx +
x(x 1)dx
0
1
Z 1
Z 2
2
=
(x x )dx +
(x2 x)dx
0

= 1/2 1/3 + 7/3 3/2


= 1,
So, E(X 2 + X) = 1 + 3/2 = 5/2.
Q2. Let the joint probability density function of random variables X and
Y be
f (x, y) = 2e(x+2y)
if x 0, y 0
= 0.
otherwise.
Find E(X) and E(Y ), and E(X 2 + Y 2 ).

15

Solution: The marginal density functions of X and Y are:


Z
2e(x+2y) dy = ex
f (x) =
Z0
2e(x+2y) dx = 2e2y ,
f (y) =
Z0
xf (x)dx
hence,
E(X) =
0
Z
=
xex dx
0

= 1,
Z
yf (y)dy
E(Y ) =
0
Z
2ye2y dy
=
0

= 1/2,
Z
Z
2
2
2
y 2 f (y)dy
x f (x)dx +
E(X + Y ) =
0
Z0
Z
=
x2 ex dx +
2y 2 e2y dx
0

= 2 + 1/2
= 5/2.
Q3. Consider repeated sampling with replacement from the box containing
five red balls, three white balls, and two blue balls. Let the sampling
continue until blue balls have shown five times. Determine the probability that, by then, red balls will have shown exactly eight times and
white balls will have shown six times.
Soluction:The samling is repeated with replacement, because it will
continue until blue balls appear five time, this imply last sample must
be a blue ball, let p1 , p2 , p3 are the probabilities of a blue ball, a red
ball and white ball appears at a sampling, respectively. Then p1 =
1/5, p2 = 1/2, p3 = 3/10 and the probability which we want to know is:
P = P (X1 = 4, X2 = 8, X3 = 6)p1
18!
=
p1 4 p2 8 p3 6 p1
4!8!6!
= 0.008.
Q4. In a drawer are 4 black socks, 6 gray socks, and 10 white socks. One
16

reaches in and randomly grabs 3 socks. Compute the probability of a


matching pair.
Soluction: Use the Hypergeometric distribution,
P (the matching pair)
= P (a pair of black socks appear) + P (a pair of gray socks appear)
+P (a pair of white socks appear)
C 2C 1
432
C 2C 1
654
C2 C1
10 9 8
= [ 4 3 16 +
] + [ 6 3 14 +
] + [ 10 3 10 +
]
C20
20 19 18
C20
20 19 18
C20
20 19 18
= 0.7895.
Q5. A father asks his sons to cut their backyard lawn. Since he does not
specify which of the three sons is to do the job, each boy tosses a coin
to determine the odd person, who must then cut the lawn. In the
case that all three get heads or tails, they continue tossing until they
reach a decision. Let p be the probability of heads and q = 1 p, the
probability of tails.
(a). Find the probability that they reach a decision in less than n
tosses.
(b). If p = 1/2, what is the minimum number of tosses required to
teach a decision with probability 0.95?
Solution: (a). Let pk is the probability of they reach a decision at kth
tossing, then this is a geometric distribution,
pk = P k1 (all three get heads or tails) P (ones result is different of others)
k1

= (p3 + q 3 )

(C31 pq 2 + C31 qp2 ),

Hence, P (reach a decision in less than n tosses)


n1
X
=
pk
k=1

1 (p3 + q 3 )n1
1 (p3 + q 3 )
= 1 (p3 + q 3 )n1 .
= (3pq 2 + 3qp2 )

(b). Since p = 1/2, we just need to know what is minimum integer


which satifying 1 ((1/2)3 + (1/2)3 )n1 0.95, this implies that n 4,
therefore, 4 is minimum number of tosses required to teach a decision
with probability 0.95.

17

Q6. If the average number of claims handled daily by an insurance company


is 5, what proportion of days have less than 3 claims? What is the
probability that there will be 4 claims in exactly 3 of the next 5 days?
Assume that the number of claims on different days is independent,
and the company insures a large number of clients, each having a small
probability of making a claim on any given day.
Solution:According the assumptions of problem, we can see the claims
process as a poisson process with with rate = 5, So
P (N < 3) = P (N = 0) + P (N = 1) + P (N = 2)
52 e5
5
5
= e + 5e +
2!
= 0.125.
Since the number of claims of different days is independent, then
P (there will be 4 claims in exactly 3 of the next 5 days)
= C53 [P (N = 4)]3 [1 P (N = 4)]2
54 e5 2
54 e5 3
) (1
)
= 10 (
4!
4!
= 0.037.
Q7. Suppose that a Scottish soldiers chest size is normally distributed with
mean 39.8 and standard deviation 2.05 inches, respectively. What is
the probability that of 20 randomly selected Scottish soldiers, exactly
five have a chest of at least 40 inches?
Solution: Let X is the random variable of soldiers chest size, and Z
is the standard normal random variable, then
P (five have a chest of at least 40 inches of 20 soldiers)
5
= C20
[P (X 40)]5 [P (X < 40)]15
40 39.8 5
X 39.8
40 39.8 15
X 39.8
5

)] [P (
<
] )
= C20
[P (
2.05
2.05
2.05
2.05
5
= C20
[P (Z 0.098)]5 [P (Z < 0.098)]15
5
= C20
0.465 0.5415
= 0.03.
Q8: According to electrical circuit theory, the voltage drop across a resistor
is related to the current flowing through the resistor by the equation
V=IR, where R is the resistance level measured in ohms, I is the current in amperes, and V is the voltage in volts. Suppose that the measured voltage in a certain electrical circuit has a normal distribution
18

with mean 120 and standard deviation 2 and five measurements of the
voltage are taken. Determine the probability that exactly two of the
measurements lie outside the range 118-122.
Solution: Let Z is the standard normal random variable, then
=
=
=
=
=

P (two of the five measurements lie outside the range 118-122)


C52 [P (X < 118 or X > 122)]2 [P (118 X 122)]3
X 120
X 120
X 120
< 1 or
> 1]2 [P (1
1)]3
C52 [P (
2
2
2
10[P (Z < 1 or Z > 1)]2 [P (1 Z 1)]3
10 (0.3174)2 (0.6826)3
0.32.

Q9. Suppose that of all the clouds that are seeded with silver iodide, 58%
show splendid growth. If 60 clouds are seeded with silver iodide,
(a). What is the probability that exactly 35 show splendid growth?
(b). What is the probability that at least 35 show splendid growth?
(c). What is the probability that the number of splendid growth in
[35,40]? (make the continuity correction of 0.5)
Solution: p = P (splendid growth) = 0.58, hence,
(a).
=

=
=
(b).

P (exactly 35 show splendid growth of 60 clouds)


35 35
C60
p (1 p)25
x 0.5 nq
x + 0.5 np
P( p
Z p
)
np(1 p)
np(1 p)
35 0.5 60 0.58
35 + 0.5 60 0.58
P(
Z
)
60 0.58 0.42
60 0.58 0.42
0.1.

P (X 35)
60
X
k k
=
C60
p (1 p)60k
k=35

35 0.5 nq
P (Z p
)
np(1 p)
35 0.5 60 0.58
= P (Z
)
60 0.58 0.42
= 0.5313.

19

(c).

P (35 X 40)
40
X
k k
=
C60
p (1 p)60k
k=35

40 + 0.5 60 0.58
35 0.5 60 0.58
Z
)
P(
60 0.58 0.42
60 0.58 0.42
= 0.4633.

20

The problems of sixth TA class of Math144


T eaching Assistant : Liu Zhi
Date : F riday, M arch 23, 2007
Q1. A statistician is trying to determine a model for claim size from historical data on a particular group of policies for an insurance company.
The statistician observes that by considering the cube of the claim sizes,
one obtains an exponential distribution with mean 2. Determine the
probability that a future claim on a policy similar to the ones being
studied will exceed 1.5.
Solution: Because X 3 have a exponential distribution with parameter
= 12 , then (X 3 /) / Weibull(, ), let = 23 , = 1/3, we know
X Weibull(23 , 1/3), therefore, the probability of it exceed 1.5 is:
1.5 1/3

P (X > 1.5) = 1 F (1.5) = e( 23 )

= 0.5642.

Q2. A particular high-technology stock whose current price is $100 either


increase 5% or decrease 3% in any given day. There is a 50% chance
that the stock price will increase on any given day independent of price
movements on previous days. Determine the probability that the stocks
price is more than five times its current price 100 trading days from
now.
Solution: Because 100(1 + 0.05)k (1 0.03)100k 500 = k 59,
this show that if the stock price is more than five times its current price
after 100 trading days, then it is increasing at least 59 days of the 100
days. Hence
P (The stocks price is more than five times its current price after 100 trading days)
X
k
=
C100
(1/2)100
59k100

59 0.5 50
)
P (Z p
100 (1/2)2
= P (Z 1.7)
= 0.0445655.
Q3. A professor takes early retirement on July 1, exactly 3 months after his
60th birthday, and purchases a term life insurance policy for $25000.
The maturity date of the policy coincides with his 65th birthday. According to a mortality table for similarly situated men, the probability that a male of exact age 60 dies before reaching his 61th birthday
21

is 0.137604. Determine the probability that the professor dies before


reaching his 61th birthday if deaths are assumed to be exponentially
distributed between ages.
Solution: Because assumed the deaths to be exponentially distributed,
i.e, let X is the lifetime of a male of exact age 60, then X exp(),
now, we need to calculate the value of , since P (X < 61|X > 60) =
1
P (X < 1) = 1 e = 0.137604 = =ln 10.137604
= 0.148039,
therefore: P (X < 0.75) = 1 e0.1480390.75 = 0.105087.

22

The problems of seventh TA class of Math144


T eaching Assistant : Liu Zhi
Date : F riday, April 13, 2007
Q1. Let X and Y be the coordinates of a random point selected uniformly
from the unit disk {(x, y) : x2 + y 2 1}. Are X and Y independent?
Are they uncorrelated? Why or why not?
Solution: The marginal density function of X is:
Z +
f (x) =
f (x, y)dy

Z +
1
=
1(x2 + y 2 1)dy

1 1
=
1( 1 x2 y 1 x2 )dy
1
Z 1x2
1
=
dy
1x2

2 1 x2
=
,

2 1y 2
Similarly, the density function of Y is g(y) =
. So,

Z yZ x
f (t, u)dtdu
P (X x, Y y) =
1 1
Z Z

1 y x
=
1( 1 u2 t 1 u2 )dtdu
1 1
Z Z 2
1 y x 1u
dtdu
=
1 1u2
=
/ P (X x)P (Y y).
This show that X and Y are not independent. And because
Cov(X, Y ) = E[XY ] E[X]E[Y ]
Z Z
p
p
1 1 1
=
xy1( 1 y 2 x + 1 y 2 )dxdy E[X]E[Y ]
1 1
Z Z 1y2
1 1
=
xydxdy E[X]E[Y ]
1 1y2
23


1 2 1y2
y( x | 2 )dy E[X]E[Y ]
2 1y
1
Z 1
2
x 1 x2 dx]2
= 0[
1
= 0.
1
=

Hence, X and Y are uncorrelated.


Q2. A right triangle has hypotenuse of length 9. If the probability density
function of one sides length is given by:
x
,
6
= 0,

if 2 < x < 4;

f (x) =

elsewhere.

What is the expected value of the length of the other side?


Solution: BecauseX 2 + Y 2 = 92 , and X has the density function f (x).

Then, E[Y ] = E[ 81 X 2 ]
Z 4
x
=
81 x2 dx
6
2
= 8.42.
Q3. An urn consists of 40 red balls and 60 green balls. What is the probability of getting exactly k red balls in a sample size of 10 if the sampling
is done (a) with replacement. (b) without replacement.
Solution: Let X is the exactly numbers of red calls in a sample size
of 10.
(a). Because sampling is done with replacement, So
4
6
k
P (X = k) = C10
( )k ( )10k .
10 10
(b). If the sampling is done without replacement, then
P (X = k) =

10k
k
C40
C60
.
10
C100

Q4. The cdf of 2v is given by, for degrees of freedom v,


Z x
2
Fv (x) = P (v x) =
fv (u)du, for x > 0
0

Show that, for v > 2, Fv (x) = Fv2 (x) 2fv (x).


24

Solution: Use the property of function: (u + 1) = u(u), then


when v > 2,
Z x
(1/2)v/2 v 1 u/2
Fv (x) =
u2 e
du
v
(
)
0
2
Z x
(1/2)v/2 v 1 u/2
= 2
u 2 de
( v2 )
0
Z x
(1/2)v/2 u/2 v 1
(1/2)v/2 v 1 x/2
2
x
e
du 2
= 2
e
+
2
( v2 )
( v2 )
0
Z x
v2
(1/2)(v2)/2 1/2 v
( 1)u 2 1 eu/2 du
= 2fv (x) + 2
v2
v2
2
( 2 ) 2
0
= Fv2 (x) 2fv (x).

Now, you can download the notes from


00
http : //ihome.ust.hk/ liuzhi/math144.html00

25

The solutions of midterm examination


T eaching Assistant : Liu Zhi
Date : F riday, April 20, 2007
Q1. What are the mean, median, and mode, respectively, for the density
1
function f (x) = (1+x
2 ) for < x < ?
(A). 0,0,0
(B). none exist
(C). not exist, not exist, 0
not exist, 0, not exist
(E).not exist, 0, 0.

(D).

Solution: Let X is a random variable with the density function, it


is easy to verify that its mean doesnt exist, and because it density
function is symmetric about 0, so both the median and mode are 0.
Q2. What is the standard deviation of the distribution: f (x) =
0, 1, 2, 3, 4?
(A). 8/3
(B). 10/3
(C). 4/5
(D). 16/81

C4x C66x
,
6
C10

x=

(E).81/25

Solution: Calculate it directly, or by the mean of hypergeometric


distrubtion.
Q3. Let X and Y be continuous random variables with joint density function:
for 0 < y < 1 |x| and 1 < x < 1;
elsewhere.

f (x, y) = 1,
= 0,

What is the variance of X?


(A). 1/18
(B). 1/6
(C). 2/9

(D). 11/18

Solution: The density funtion of X is:


Z 1|x|
dx
f (x) =
0

= 1 x, 1 < x < 1.
So,

EX =

xf (x)dx
1
Z 0

x(1 x)dx

x(1 + x)dx +
0

1
1
1
1 2 0
x |1 + x3 |01 + x2 |10 + x3 |10
2
3
2
3
= 0,
=

26

(E). 2/3

EX

x (1 + x)dx +

=
1

x2 (1 x)dx

= 1/3 1/4 + 1/3 1/4


= 1/6.
var(X) = EX 2 (EX)2 = 1/6.
Q4. Let X and Y be continuous random variables with joint density function:
3
f (x) =
(2 x y),
for 0 < x < 2, 0 < y < 2, and x + y < 2,
4
= 0,
elsewhere.
What is the conditional probability P (X < 1|Y < 1)?
(A). 1/2

(B). 3/4

(C). 49/64

(D). 6/7

(E).7/8

Solution: The density function of Y is


Z 2
3
(2 x y)1(x + y < 2)dx
f (y) =
0 4
Z
3 2y
=
(2 x y)dx
4 0
3
1
=
[4 2y x2 |02y y(2 y)]
4
2
3
3
3
3
= 3 y (2 y)2 y + y 2
2
8
2
4
3 2 3
3
=
y y+ .
8
2
2
1

3
3
3
( y 2 y + )dy
2
2
0 8
= 7/8.

So:

P (Y < 1) =

P (X < 1, Y < 1) =
0

3
(2 x y)1(x + y < 2)dxdy
4

= 6/8,
Hence,
P (X < 1, Y < 1)
P (Y < 1)
= 6/7.

P (X < 1 | Y < 1) =

27

Q5. Let X and Y be independent random variables with binomial distributions Bin(k; n, a) and Bin(k; n, b) respectively. Let Z = X + 2Y .
Then for all  > 0, Tchebycheffs inequality guarantees that P (|Z
na 2nb| ) is always less than or equal to what?
(A). (na(1 a) + nb(1 b))/2 .
(B). (na(1 a) + 2nb(1 b))/2 .
(C). (na(1-a)+4nb(1-b))/2 .
(D). (na(1 a) + 2nb(1 b))/3 .
(E). (na(1 a) + 4nb(1 b))/3 .
Solution: Recalled that the Tchebycheffs inequality is:
P (|X EX| )

V ar(X)
.
2

Because V ar(Z) = V ar(X) + V ar(2Y ) = na(1 a) + 4nb(1 b), So


P (|Z na 2nb| ) (na(1 a) + 4nb(1 b))/2 .
Q6. Simplify the expression of

PN

k=n

Ckn .

Solution: Because:
k!
n!(k n)!
k!(k + 1 k + n)
=
(n + 1)!(k n)!
(k + 1)!
k!
=

(n + 1)!(k n)! (n + 1)!(k n 1)!


n+1
= Ck+1
Ckn+1 , k = n + 1, n + 1, .

Ckn =

So,

N
X

Ckn

k=n
n
= Cnn + Cn+1
+ + CNn
n+1
n+1
n+1
n+1
n+1
n+1
n+1
= 1 + Cn+1+1
Cn+1
+ Cn+2+1
Cn+2
+ + CNn+1
1+1 CN 1 + CN +1 CN
= CNn+1
+1 .

Q7. A man invites his fiancee to a fine hotel for a Sunday brunch. They
decide to meet in the lobby of the hotel between 11 : 30 am. and 12 : 00
moon. If they arrive at random times during this period, what is the
probability that they will meet within 10 minutes?

28

Solution: Let X and Y is the time of the man and his fiancee arrive
at the hotel, start from 11 : 30, then X and Y are independent and
have identical density function:
1
,
30
= 0,

f (x) =

Then,
=
=
=
=
=
=
=
=
=

if 0 < x < 30,


elsewhere.

P (|X Y | 10)
P (10 X Y 10)
Z +
P (10 X Y 10 | Y = y)f (y)dy

Z 30
P (10 X Y 10 | Y = y)f (y)dy
0
Z 30
1
P (y 10 X 10 + y | Y = y)dy
30 0
Z 30 Z 30(y+10)
1
1
dy
30 0
30
0(y10)
Z 30 Z 30
Z 20 Z y+10
Z 10 Z y+10
1
dy]
dy +
dy +
[
900 0
20
y10
10
y10
0
Z 30
Z 20
Z 10
1
(40 y)dy]
20dy +
(y + 10)dy +
[
900 0
20
10
1
[100 + 50 + 600 250]
900
5
.
9

Q8. Two subcontractors have been hired to build different parts of an experimental aircraft. The expected completion times for the two subcontractors are 10 months and 12 months, respectively. Both parts are
necessary before final assembly of the aircraft can take place. How long
should the primary contractor expect to wait to complete final assembly if at least one of the subcontractors is expected to be finished in 6
months?
Solution: Because E[Min(X,Y)]=6, so E[Max(X,Y)]=E[X]+E[Y]-E[Min(X,Y)]=16.
Q9. Calculate the expected number of aces in a randomly selected five cards
from a pool of 52 card that is found to have exactly twol jacks.

29

Solution: Let X is the number of aces in five cards and the set A =
{It have exactly two jacks in these 5 cards}. Then
E[X | A]
= 0 P (X = 0 | A) + 1 P (X = 1 | A) + 2 P (X = 2 | A) + 3 P (X = 3 | A)
P (A, X = 1) 2P (A, X = 2) 3P (A, X = 3)
=
+
+
,
P (A)
P (A)
P (A)
We calculate the needed probability as following:
3
C42
C48
P (A) =
,
5
C52
2
C44
C42 C41
,
5
C52
C 1 C 2C 2
P (A, X = 2) = 44 54 4 ,
C52
3 2
C C
P (A, X = 3) = 4 5 4 ,
C52

P (A, X = 1) =

Therefore,

E[X | A]
1
1
=
[C 2 C 2 C 1 + 2C44
C42 C42 + 3C43 C42 ]
3
C48 C42 44 4 4
1
=
.
4

Q10. (a.) Given X is a normal variable with mean equals 2 and variance
equals 4, compute P (X 2 X < 6).
N (0, 1). Now we can compute
Solution: X N (2, 4) implies X2
2
2
the probability: P (X X < 6)
= P ((X + 2)(X 3) < 0)
= P (2 < X < 3)
2 2
X 2
32
= P(
<
<
)
2
2
2
= 0.6687.
(b.) Given X is a normal random variable with mean equals 1 and
variance equals 2; Y is a normal random variable with mean equals 3
and variance equals 4. Both X and Y are independent of each other.
Compute P (2X + 3Y > 9).
30

Solution: Because X N (1, 2), Y N (3, 4), then 2X N (2, 8), Y


N (9, 36), also since X and Y are independent, hence 2X + 3Y
N (11, 44). and let Z is the standard normal variable, Then
P (2X + 3Y > 9)
1
= P (Z > )
11
= P (Z < 0.3)
= 0.6179.

31

The problems of eighth TA class of Math144


T eaching Assistant : Liu Zhi
Date : F riday, April 27, 2007
Q1. Civil engineers believe that W, the amount of weight(in units of 1,000
pounds) that a certain span of a bridge can withstand without structural damage resulting is normally distributed with mean 400 and standard deviation 40. Suppose that the weight of a car is a random variable
with mean 3 and standard deviation 0.3. How many cars would have
to be on the bridge span for the probability of structural damage to
exceed 0.1?
Solution: Let X is random variable of weight that a certain span of a
bridge can withstand without structural damage resulting(in units of
1,000 P
pounds), X N (400, 402 ) and Yi is the weigth
Pn of a car, then by
n
CLT, i=1 Yi N (30n, 0.09n), this imply that i=1 Yi X N (30n
400, 1600 + 0.09n), therefore the probability of structural damage when
n cars on the bridge span:
n
X
p = P(
Yi > X)
i=1
n
X
= P(
Yi X > 0)
i=1
P
( ni=1 Yi X) (30n 400)
0 (30n 400)

>
)
= P(
1600 + 0.09n
1600 + 0.09n
400 30n
),
= P (Z >
1600 + 0.09n

p > 0.1 imply n 16 by check the standard normal distribution table.


Q2. If T
(a).
(b).
(c).

has a tdistibution with 8 degrees of freedom, find


P (T 1),
P (T 2),
P (1 < T < 1).

Solution: (a). Check the tdistibution table, we find P (t8 0.889) =


0.20, P (t8 1.108) = 0.15,then
P (T 1)
= 0.20 + (0.15 0.20)
= 0.174658.
32

1 0.889
1.108 0.889

(b). Similarly, because P (t8 1.86) = 0.05, P (t8 2.306) = 0.025, so


P (T 2)
= 1 P (T > 2)
= 1 [0.05 + (0.025 0.05)

2 1.86
]
2.306 1.86

= 0.957848.
(c).
=
=
=
=

P (1 < T < 1)
P (T < 1) P (T < 1)
1 P (T 1) P (T 1)
1 2 0.174658
0.650684.

Q3. (a). Find a 95% confident interval for the mean of a normal distribution with unknown variance based fon the 20 samples data: (7, 9, 3, 2,
3, 8, 4, 6, 2, 6, 4, 3, 8, 3, 2, 7, 9, 5, 8, 8).
(b). Find the confidence coefficient (1-) of the tinterval (3.738,
6.962) about population mean ().
Solution: (a). Because the variance is unknown, we should use the
testimation:
q P
P20
1
1
2
I. Two tails: x = 20 i=1 xi = 5.35, s =
x2 ) =
( 20
i=1 xi 20
19
2.5188. So, in this case, the confident interval of mean() with confident coefficient 0.95 is [5.35 t0.05/2,19 s20 , 5.35 + t0.05/2,19 s20 ] = [5.35
2.9032.5188

, 5.35 + 2.9032.5188
] = [3.71497, 6.98503].
20
20
II. One tail: the upper one-sided bound is:
1.729 2.5188
s

x + t0.05,19 = 5.35 +
= 6.32381,
20
20
and the lower one-sided bound is:
s
1.729 2.5188

x t0.05,19 = 5.35
= 4.37619.
20
20
(b). 5.35 + t/2,19 s20 = 6.962 = t/2,19 = 2.86211 = = 0.01.
Q4. Find a 95% confident interval for 1 2 with x1 = 10, x2 = 9, s21 =
9, s22 = 4, and n1 = n2 = 100 when sampling from two independent
normal distribution.
33

Solution: Caseq
I: Suppose that two normal distributions have same
(n1 1)s21 +(n2 1)s22
variances, sp =
= 2.55, then the 95% confident inn1 +n2 2
terval for 1 2 is:
r
r
1
1
1
1
[
x1 x2 t/2,n1 +n2 sp
+ , x1 x2 + t/2,n1 +n2 sp
+ ]
n1 n2
n1 n2
= [0.293178, 1.70682].
Case II. When two normal distributions have different variances, v =
s2

s2

( n1 + n2 )2
1

2
s2
s2
( n1 )2
( n2 )2
1
2
+
n1 1
n2 1

= 172. then in this case, the 95% confident interval for

1 2 is:
[
x1 x2 t/2,v

s11

n1
= [0.293312, 1.70669].

s22
n2

s
, x1 x2 + t/2,v

s2
s11
+ 2]
n1 n2

Q5. Find a 95% confident interval for 1 2 with x1 = 10, x2 = 9, s21 =


9, s22 = 4, and n1 = n2 = 10 in sampling from two independent normal
distributions with same but unknown variances.
Solution: We can obtain the answer by change the n1 , n2 of Q4 from
100,100 to 10,10, respectively. Then sp = 2.55, same as Q4. So the
95% confident interval for 1 2 is [0.0671124, 1.93289].

34

The problems of ninth TA class of Math144


T eaching Assistant : Liu Zhi
Date : F riday, M ay 4, 2007
P
0
2 . Show that
Q1. Let us define S 2 = n1 ni=1 (Xi X)
0

E(S 2 ) = [(n 1)/n] 2 ,


0

and hence S 2 is a biased estimator for 2 .


P
0
2=
Solution: Because S 2 = n1 ni=1 (Xi X)

1
n

Pn

i=1

2 , so
Xi2 X

1X 2
2]
E(S ) = E[
X ] E[X
n i=1 i
02

1X
1X
=
E[Xi2 ] E(
Xi )2
n i=1
n i=1
n
n
1 X
2 X
1X
2
E[Xi ] 2
E[Xi2 ] 2
E[Xi Xj ]
=
n i=1
n i=1
n i<j

n1
n1
E[X 2 ]
[EX]2
n
n
n1
(E[X 2 ] [EX]2 )
=
n
n1 2
=
.
n

This is required result and hence, S 2 is a biased estimator for 2 .


Q2. If X is a binomial random variable, show that
(a). p = X/n is an unbiased estimator of p;
n/2

is a biased estimator of p; (c). Show that the estima(b). p0 = X+


n+ n
0
tor p becomes unbiased as n .
Solution: (a). E[
p] = E[X]/n = np/n = p. This show that p is an
unbiased estimator of p.

n/2
np+ n/2

(b). E[p0 ] = E[X]+


=
=
/p, so p0 is a biased estimator of p.
n+ n
n+ n

np + n/2

(c).
limn
n+ n
p
1
+ limn

= limn
n
2(1 + n)
1+ n
= p.
35

so, the required result follows.


Q3. An electrical firm manufactures light bulbs that have a length of life
that is approximately normally distributed with a standard deviation
of 40 hours. If a sample of 30 bulbs has an average life of 780 hours,
(a). Find a 96% confidence interval for population mean of all bulbs
produced by this firm;
(b). How large a sample is needed if we wish to be 96% confident that
our sample mean will be within 10 hours of the true mean?
Solution: The sample mean x is 780, and the standard deviation of
population is 40, using the Normal table, we can get z0.02 = 2.05. Hence
the 96% confidence interval for is:

[
x z0.02 ( ), x + z0.02 ( )] = [765.029, 794.971].
30
30
(b). We can be 96% confident that this error will less than t/2 ( sn ),
i.e,
z0.02 40 2
n=(
) = 67.24
e
Therefore, a sample with size 68 is needed if we wish be 96% confident
that our sample mean will be within 10 hours of the true mean.
Q4. Many cardiac patients wear implanted pacemakers to control their hearbeat. A plastic connector module mounts on the top of the pacemaker.
Assuming a standard deviation of 0.0015 and an approximate normal
distribution,
(a). Find a 95% confidence interval for the mean of all connector modules made by a certain manufacturing company. A random sample of
75 modules has an average of 0.310 inch.
(b). How large a sample is needed if we wish to be 95% confident that
our sample mean will be within 0.0005 inch of true mean?
Solution: (a).From the problem, we know:
n = 75, x = 0.310, = 0.0015,
hence the 95% confidence interval for the mean of all modules is:
0.0015
0.0015
), 0.310 + z0.025 (
)] = [0.309661, 0.310339].
[0.310 z0.025 (
75
75
(b).
n=(

z0.025 0.0015 2
) = 35.8225.
0.0005
36

Therefore, a sample with size 38 is needed if we wish to be 95% confident


that our sample mean will be within 0.0005 inch of true mean.
Q5. The heights of a random sample of 50 college students showed a mean
of 174.5 centimeters and a standard deviation of 6.9 centimeters.
(a). Construct a 98% confidence interval for the mean height of all
college students. (b). What can we assert with 98% confidence about
the possible size of our error it we estimate the mean height of all
college students to be 174.5 centimeters?
Solution: Because standard deviation of the population is unknown,
so we should use the tdistribution:
(a). s = 6.9, x = 174.5, n = 50, so 98% confidence interval for the mean
height of all college students is:
s
6.9
[174.5 t0.01,49 ( ), 174.5 + t0.01,49 ( )] = [173.229, 175.771].
50
50
(b). We can be 98% confident that this error will less than t/2 ( sn ),
i.e,
6.9
e t0.01,49 ( ) = 1.27148.
50
Q6. A random sample of 100 automobile owners shows that, in the state
of Virginia, an automobile is driven on the average 23,500 kilometers
per year with a standard deviation of 3900 kilometers. Assume the
distribution of measurements to be approximately normal.
(a). Construct a 99% confidence interval for the average number of
kilometers an automobile is driven annually in Virginia.
(b). What can we assert with 99% confidence about the possible size
of our error if we estimate the average number of kilometers dirven by
car owners in Virginia to be 23,500 kilometers per year?
Solution: We use the tdistribution, x = 23, 500, s = 3900, n = 100,
then the 99% confidence interval for population mean is:
3900
3900
), 23500 + t0.005,99 (
)] = [22479.4, 24520.6].
[23500 t0.005,99 (
100
100
(b).
3900
e t0.005,99 (
) = 1020.63.
100

37

Q7. An efficiency expert wishes to determine the average time that it takes
to drill three holes in a certain metal clamp. How large a sample will
he need to be 95% confident that his sample mean will be within 15
seconds of the true mean? Assume that it is known from previous
studies that = 40 seconds.
Solution: The population standard deviation is = 40, Then by
Theorem 9.2 of textbook,
n=(

(1.96)(40) 2
) = 27.318,
15

Therefore, a sample with size 28 be needed to be 95% confident that


his sample mean will be within 15 seconds of the true mean.

38

The problems of Tenth TA class of Math144


T eaching Assistant : Liu Zhi
Date : F riday, M ay 11, 2007
Q1. A manufacturer of car batteries claims that his batteries will last, on
average, 3 years with a variance of 1 year. If 5 of this batteries have
lifetimes of 1.9, 2.4, 3.0, 3.5 and 4.2 years, construct a 95% confidence
interval for 2 and decide if the manufacturers claim that 2 = 1
is valid. Assume the population of battery lives to be approximately
normally distributed.
2

Solution: Because P ( (n1)S


< 2 <
2
/2

n
1 X
S =
(xi
x)2 =
n 1 i=1
2

Pn

i=1

(n1)S 2
)
21/2

= 1, n = 5, = 0.05,

P
x2i ( ni=1 xi )2
(5)(48026) 225
=
= 0.815,
n(n 1)
20

v = 4, 20.025 = 11.143, 210.025 = 0.484, then the 95% confidence interval for 2 is:
[

(n 1)S 2 (n 1)S 2
,
] = [0.29256, 6.73554],
2/2
21/2

since 1 belong in the interval, so the claim that 2 = 1 is valid.


Q2. A random sample of 20 students obtained a mean of x = 72 and a variance of S 2 = 16 on a college placement test in mathematics. Assuming
the scores to be normally distributed, contruct a 98% confidence interval for 2 .
Solution: Similar calculation steps as last question, n = 20, S 2 =
16, v = 19, 20.01 = 36.191, 210.01 = 7.633, so the 98% confidence interval for 2 is:
(n 1)S 2 (n 1)S 2
,
] = [8.39988, 39.8271].
[
2/2
21/2
Q3. A random sample of 64 bags of white cheddar popcorn weighed, on
average, 5.23 ounces with a standard deviation of 0.24 ounces. Test
the hypothesis that = 5.5 ounces against the alternative hypothesis,
< 5.5 ounces at the 0.05 level of significance.
Solution: 1. H0 : = 5.5 ounces.
39

2. H1 : < 5.5 ounces.


3. = 0.05.
4. Critical region: z < 1.645.
5. Computations: x = 5.23 ounces, = 0.24 ounces, and
z=

5.235.5

0.24/ 64

= 9.

6. Decision: Reject H0 , accept H1 .


Q4. Past experience indicated that the time for high school seniors to complete a standardized test is a normal random variable with a mean of
35 minutes. If a random sample of 20 high school seniors took an average of 33.1 minutes to complete this test with a standard deviation
of 4.3 minutes, test the hypothesis at the 0.05 level of significance that
= 35 minutes against the alternative that < 35 minutes.
Solution: 1. H0 : = 35 minutes.
2. H1 : < 35 minutes.
3. = 0.05.
4. Critical region: z < 1.645.
5. Computations: x = 33.1 minutes, = 4.3 minutes, and
z=

33.135

4.3/ 20

= 1.97606.

6. Decision: Reject H0 , accept H1 .

40

You might also like